Vector calculus double integrals

Click For Summary
The discussion focuses on calculating the volume V of the solid under the surface z=4-x^2-y^2 over the rectangle defined by 0<=x<=1 and 0<=y<=2. The initial double integral setup, ∫∫(4-x^2-y^2) dx dy, is confirmed to be correct for this rectangular region. There is no need to convert to polar coordinates, as that would only be suitable for circular regions. The general formula for volume, ∫∫ f(x,y) dA, is acknowledged as valid but unnecessary for this specific problem. The conversation emphasizes sticking with the rectangular limits for accurate volume calculation.
braindead101
Messages
158
Reaction score
0
Find the volume V of the solid under the surface z=4-x^2-y^2 and over the rectangle R consisting of all points (x,y) such that 0<=x<=1 and 0<=y<=2.

I have started, but am unsure if my approach is correct or not.

x = 4-x^2-y^2
\int^{2}_{0}\int^{1}_{0} 4-x^{2}-y^{2} dx dy
is this correct?

or should i be putting it in polar coordinates, in which case how do i set up the limit of integration
\int\int_{\Omega} f(x,y) dA
\Omega = \left\{ (x,y) | 0&lt;=x&lt;=1, 0&lt;=y&lt;=2 \right\} ? is this right?
 
Physics news on Phys.org
You're looking for the volume under the surface, within the bounds of the rectangle, right? Your first one is fine, though I think you meant to say f(x,y) = 4-x^2-y^2.
 
braindead101 said:
Find the volume V of the solid under the surface z=4-x^2-y^2 and over the rectangle R consisting of all points (x,y) such that 0<=x<=1 and 0<=y<=2.

I have started, but am unsure if my approach is correct or not.

x = 4-x^2-y^2
I'm not sure why you would write that: z= 4- x2- y2, not x.

\int^{2}_{0}\int^{1}_{0} 4-x^{2}-y^{2} dx dy
is this correct?

or should i be putting it in polar coordinates, in which case how do i set up the limit of integration
\int\int_{\Omega} f(x,y) dA
\Omega = \left\{ (x,y) | 0&lt;=x&lt;=1, 0&lt;=y&lt;=2 \right\} ? is this right?
Of course, your first calculation is correct. In fact, the second formula,
\int\int_{\Omega} f(x,y) dA
is also correct- it's just a general statement of the formula for volume.
But there is no reason to change to polar coordinates. Polar coordinates would be appropriate if you were finding the volume over a disk.
 
Question: A clock's minute hand has length 4 and its hour hand has length 3. What is the distance between the tips at the moment when it is increasing most rapidly?(Putnam Exam Question) Answer: Making assumption that both the hands moves at constant angular velocities, the answer is ## \sqrt{7} .## But don't you think this assumption is somewhat doubtful and wrong?

Similar threads

Replies
4
Views
2K
Replies
20
Views
2K
  • · Replies 19 ·
Replies
19
Views
2K
  • · Replies 10 ·
Replies
10
Views
2K
Replies
12
Views
2K
  • · Replies 6 ·
Replies
6
Views
2K
  • · Replies 2 ·
Replies
2
Views
2K
  • · Replies 1 ·
Replies
1
Views
2K
Replies
6
Views
2K
  • · Replies 14 ·
Replies
14
Views
2K